Diễn Đàn MathScopeDiễn Đàn MathScope
  Diễn Đàn MathScope
Ghi Danh Hỏi/Ðáp Thành Viên Social Groups Lịch Ðánh Dấu Ðã Ðọc

Go Back   Diễn Đàn MathScope > Sơ Cấp > Việt Nam và IMO > 2013

News & Announcements

Ngoài một số quy định đã được nêu trong phần Quy định của Ghi Danh , mọi người tranh thủ bỏ ra 5 phút để đọc thêm một số Quy định sau để khỏi bị treo nick ở MathScope nhé !

* Nội quy MathScope.Org

* Một số quy định chung !

* Quy định về việc viết bài trong diễn đàn MathScope

* Nếu bạn muốn gia nhập đội ngũ BQT thì vui lòng tham gia tại đây

* Những câu hỏi thường gặp

* Về việc viết bài trong Box Đại học và Sau đại học


Trả lời Gởi Ðề Tài Mới
 
Ðiều Chỉnh Xếp Bài
Old 24-07-2013, 11:19 PM   #1
novae
+Thành Viên Danh Dự+
 
novae's Avatar
 
Tham gia ngày: Jul 2010
Đến từ: Event horizon
Bài gởi: 2,453
Thanks: 53
Thanked 3,057 Times in 1,288 Posts
[IMO 2013] Bài 4 - Hình học phẳng

Cho tam giác nhọn $ABC$ với trực tâm $H$. Cho $W$ là một điểm tùy ý trên cạnh $BC$, khác với các điểm $B$ và $C$. Các điểm $M$ và $N$ tương ứng là chân các đường cao hạ từ $B$ và $C$. Kí hiệu $\omega_1$ là đường tròn ngoại tiếp tam giác $BWN$, và gọi $X$ là điểm trên $\omega_1$ sao cho $WX$ là đường kính của $\omega_1$. Tương tự, kí hiệu $\omega_2$ là đường tròn ngoại tiếp tam giác $CWM$, và gọi $Y$ là điểm trên $\omega_2$ sao cho $WY$ là đường kính của $\omega_2$. Chứng minh rằng các điểm $X,Y$ và $H$ thẳng hàng.
[RIGHT][I][B]Nguồn: MathScope.ORG[/B][/I][/RIGHT]
 
__________________
M.

thay đổi nội dung bởi: novae, 25-07-2013 lúc 01:55 AM
novae is offline   Trả Lời Với Trích Dẫn
The Following 2 Users Say Thank You to novae For This Useful Post:
Raul Chavez (01-08-2013), thaygiaocht (27-07-2013)
Old 25-07-2013, 02:51 AM   #2
hansongkyung
+Thành Viên+
 
Tham gia ngày: Jan 2012
Đến từ: Han Tae Woong - IMO 1998
Bài gởi: 493
Thanks: 109
Thanked 417 Times in 241 Posts
Gửi tin nhắn qua Yahoo chát tới hansongkyung
Mình làm thế này không biết có đúng không.

Theo định lý Miquel ta có: 3 đường tròn $\omega_1 ; \omega_2; (AMN)$ cùng đi qua 1 điểm. Gọi điểm đó là $F$

Và ta cùng được là 3 điểm $X; F; Y$ thẳng hàng.

$O_1;O_2$ lần lượt là tâm của 2 đường tròn $\omega_1 ; \omega_2$ khi đó $O_1O_2 \parallel XY$ (đường trung bình).

Và $O_1O_2 \perp AW$ vì $AW$ là trục đẳng phương của $\omega_1 ; \omega_2$

Lại có: $HF \perp AW$ vì $AH$ là đường kính của $(AMN)$.

$\Rightarrow HF \parallel O_1O_2$

Vậy $H,F, X, Y$ thẳng hàng.

Không biết mình có sai ở đâu không. Tại vì có ai đã nói rằng
Trích:
Nếu bạn tìm ra một lời giải quá dễ dàng thì bạn đã sai

[RIGHT][I][B]Nguồn: MathScope.ORG[/B][/I][/RIGHT]
 
Hình Kèm Theo
Kiểu File : jpg P3.ggb.jpg (17.1 KB, 385 lần tải)

thay đổi nội dung bởi: hansongkyung, 25-07-2013 lúc 02:54 AM
hansongkyung is offline   Trả Lời Với Trích Dẫn
Old 25-07-2013, 07:09 AM   #3
einstein1996
Senior Member
 
Tham gia ngày: Nov 2011
Đến từ: việt nam
Bài gởi: 103
Thanks: 77
Thanked 43 Times in 28 Posts
Trích:
Nguyên văn bởi hansongkyung View Post
Mình làm thế này không biết có đúng không.

Theo định lý Miquel ta có: 3 đường tròn $\omega_1 ; \omega_2; (AMN)$ cùng đi qua 1 điểm. Gọi điểm đó là $F$

Và ta cùng được là 3 điểm $X; F; Y$ thẳng hàng.

$O_1;O_2$ lần lượt là tâm của 2 đường tròn $\omega_1 ; \omega_2$ khi đó $O_1O_2 \parallel XY$ (đường trung bình).

Và $O_1O_2 \perp AW$ vì $AW$ là trục đẳng phương của $\omega_1 ; \omega_2$

Lại có: $HF \perp AW$ vì $AH$ là đường kính của $(AMN)$.

$\Rightarrow HF \parallel O_1O_2$

Vậy $H,F, X, Y$ thẳng hàng.

Không biết mình có sai ở đâu không. Tại vì có ai đã nói rằng


Bạn Hùng coi lại xem phải là $FW$ là trục đẳng phương của $\omega_1 ; \omega_2$ như vậy thì ta chỉ có $O_1O_2 \perp FW$ và $HF \perp AW$. Phải chứng minh được A, F, W thảng hàng.
[RIGHT][I][B]Nguồn: MathScope.ORG[/B][/I][/RIGHT]
 
einstein1996 is offline   Trả Lời Với Trích Dẫn
Old 25-07-2013, 08:18 AM   #4
12121993
+Thành Viên+
 
Tham gia ngày: Feb 2012
Bài gởi: 81
Thanks: 23
Thanked 70 Times in 41 Posts
Áp dụng định lý Miquel cho tam giác $ABC$, với 3 điểm $W, M, N$ ta có $\omega_1$ , $\omega_2$ , $(AMN)$ đồng quy tại $D$.
Dễ thấy $(AMN)$ cũng chính là đường tròn đường kính $AH$.
Ta có $AN.AB=AM.AC$ hay $A$ có cùng phương tích đối với $\omega_1$ , $\omega_2$. Do đó, nó phải thuộc trục đẳng phương $WD$ của hai đường tròn.
Ta có $DX, DY$ vuông góc với $WD$, $DH$ vuông góc với$DA$. Do đó, $X,Y,H$ thằng hàng.
[RIGHT][I][B]Nguồn: MathScope.ORG[/B][/I][/RIGHT]
 
Hình Kèm Theo
Kiểu File : png IMO 2013 Pro5.png (31.2 KB, 22 lần tải)
12121993 is offline   Trả Lời Với Trích Dẫn
Old 25-07-2013, 11:58 AM   #5
hansongkyung
+Thành Viên+
 
Tham gia ngày: Jan 2012
Đến từ: Han Tae Woong - IMO 1998
Bài gởi: 493
Thanks: 109
Thanked 417 Times in 241 Posts
Gửi tin nhắn qua Yahoo chát tới hansongkyung
Trích:
Nguyên văn bởi einstein1996 View Post
Bạn Hùng coi lại xem phải là $FW$ là trục đẳng phương của $\omega_1 ; \omega_2$ như vậy thì ta chỉ có $O_1O_2 \perp FW$ và $HF \perp AW$. Phải chứng minh được A, F, W thảng hàng.
Ta có $AN \cdot AC = AM \cdot AB$ vì $\bigtriangleup AMN \sim \bigtriangleup ABC$ nên $A$ thuộc trục đẳng phương của 2 đường tròn $\omega_1; \omega_2$

Cám ơn bạn, mình sẽ edit lại cho hoàn chỉnh hơn

[RIGHT][I][B]Nguồn: MathScope.ORG[/B][/I][/RIGHT]
 

thay đổi nội dung bởi: hansongkyung, 25-07-2013 lúc 12:03 PM
hansongkyung is offline   Trả Lời Với Trích Dẫn
Old 25-07-2013, 12:49 PM   #6
n.v.thanh
Moderator
 
n.v.thanh's Avatar
 
Tham gia ngày: Nov 2009
Bài gởi: 2,849
Thanks: 2,980
Thanked 2,537 Times in 1,008 Posts
Hùng Chở Lợn thì ai mà chả từng nghe tên . Năm nay có 2 bài hình, dự là đoàn VN lại top 10
[RIGHT][I][B]Nguồn: MathScope.ORG[/B][/I][/RIGHT]
 
n.v.thanh is offline   Trả Lời Với Trích Dẫn
The Following User Says Thank You to n.v.thanh For This Useful Post:
hoangnam94 (25-07-2013)
Old 27-07-2013, 02:04 PM   #7
thaygiaocht
+Thành Viên+
 
thaygiaocht's Avatar
 
Tham gia ngày: Aug 2012
Đến từ: Chuyên Hà Tĩnh
Bài gởi: 165
Thanks: 793
Thanked 216 Times in 93 Posts
Trích:
Nguyên văn bởi novae View Post
Cho tam giác nhọn $ABC$ với trực tâm $H$. Cho $W$ là một điểm tùy ý trên cạnh $BC$, khác với các điểm $B$ và $C$. Các điểm $M$ và $N$ tương ứng là chân các đường cao hạ từ $B$ và $C$. Kí hiệu $\omega_1$ là đường tròn ngoại tiếp tam giác $BWN$, và gọi $X$ là điểm trên $\omega_1$ sao cho $WX$ là đường kính của $\omega_1$. Tương tự, kí hiệu $\omega_2$ là đường tròn ngoại tiếp tam giác $CWM$, và gọi $Y$ là điểm trên $\omega_2$ sao cho $WY$ là đường kính của $\omega_2$. Chứng minh rằng các điểm $X,Y$ và $H$ thẳng hàng.
Cũng như APMO 2013, IMO 2013 có 2 bài hình trong đó có 1 bài hình ở mức độ dễ.
1. Trước hết nhận xét $X, Y, Z $ thẳng hàng; $A, Z, W $ thẳng hàng và $XY $ vuông góc $AW. $
2. Ta có $\widehat{NHM}=\widehat{NZM}(=180^0-\widehat{A}) $ nên $N, H, Z, M $ đồng viên, khi đó $HZ $ vuông góc $AW. $
Từ đó bài toán được chứng minh.
[RIGHT][I][B]Nguồn: MathScope.ORG[/B][/I][/RIGHT]
 

thay đổi nội dung bởi: thaygiaocht, 27-07-2013 lúc 02:07 PM
thaygiaocht is offline   Trả Lời Với Trích Dẫn
Trả lời Gởi Ðề Tài Mới

Bookmarks

Ðiều Chỉnh
Xếp Bài

Quuyền Hạn Của Bạn
You may not post new threads
You may not post replies
You may not post attachments
You may not edit your posts

BB code is Mở
Smilies đang Mở
[IMG] đang Mở
HTML đang Tắt

Chuyển đến


Múi giờ GMT. Hiện tại là 05:49 PM.


Powered by: vBulletin Copyright ©2000-2024, Jelsoft Enterprises Ltd.
Inactive Reminders By mathscope.org
[page compression: 66.28 k/75.08 k (11.73%)]